Proof for interval/function boundaries.

  • Thread starter paalfis
  • Start date
  • Tags
    Proof
In summary, the conversation discusses the proof that the image of a function and a semigroup are not lower bounded. For the semigroup, it is shown that if an infimum M exists, then 1/-x is also a member of the semigroup and is smaller than M, contradicting the existence of M as a lower bound. For the function, it is proven that there exists a value x such that f(x) is equal to the lower bound M-1, showing that M cannot be a lower bound. The use of discriminants and quadratic equations is mentioned as a possible approach for proving this.
  • #1
paalfis
69
2

Homework Statement


Prove that the image of f(x)=-x2+2x+1 and the semigroup {x-1 : x<0} are not lower bounded.
(I do not know the exact translation to english for this last expression, I think they mean that there is not lower limit)

The Attempt at a Solution


For the case of the semigroup, I thought of something like this (I am just starting with mathematical formal proofs): Suppose (to look for a contradiction) that there is an infimum M, which must exist if the semigroup has a lower bound, because of the continuity of real numbers. Then M<1/x for all x real <0, and M>M' for all M' smaller than the semigroup.
Now if I multiply x<0 by a number between 0 and 1, I get another real number "y" smaller than "x" that is also real and <0; so that 1/y belongs to the semigroup and 1/y>M, so M is not a lower bound, this contradicts the first proposition, so that M does not exist.

I have the feeling that there is something wrong with this, specially in the part of multiplying by another number... help!

For the case of the function I tried to do something similar, first by proving that f(x+1)<f(x) for all x>1/4, but this just prove that the function is monotonically decreasing after x=1/4, it does not prove the absence of a lower limit.
 
Physics news on Phys.org
  • #2
paalfis said:

Homework Statement


Prove that the image of f(x)=-x2+2x+1 and the semigroup {x-1 : x<0} are not lower bounded.
(I do not know the exact translation to english for this last expression, I think they mean that there is not lower limit)


The Attempt at a Solution


For the case of the semigroup, I thought of something like this (I am just starting with mathematical formal proofs): Suppose (to look for a contradiction) that there is an infimum M, which must exist if the semigroup has a lower bound, because of the continuity of real numbers. Then M<1/x for all x real <0, and M>M' for all M' smaller than the semigroup.
Now if I multiply x<0 by a number between 0 and 1, I get another real number "y" smaller than "x" that is also real and <0; so that 1/y belongs to the semigroup and 1/y>M, so M is not a lower bound, this contradicts the first proposition, so that M does not exist.

I have the feeling that there is something wrong with this, specially in the part of multiplying by another number... help!

There is an easier way: Suppose [itex]M[/itex] is a lower bound. Then [itex]M < 0[/itex], so [itex]1/(2M) < 0[/itex] also.

The easiest way is to observe that, by definition of multiplicative inverse, the map [itex]x \mapsto x^{-1}[/itex] is a surjection from the strictly negative reals to the strictly negative reals, which don't have a lower bound.

For the case of the function I tried to do something similar, first by proving that f(x+1)<f(x) for all x>1/4, but this just prove that the function is monotonically decreasing after x=1/4, it does not prove the absence of a lower limit.

Suppose [itex]M[/itex] is a lower bound on the image of [itex]f[/itex]. Then [itex]M \leq f(3) = -7 < 0[/itex]. Since [itex]M < 0[/itex] is a lower bound, there should be no real [itex]x[/itex] for which [itex]f(x) = M - 1[/itex]. Is that the case? [Hint: think about discriminants of quadratic equations.]

(This does rely on the fact that for every positive real [itex]c[/itex] there exists a positive real [itex]x[/itex] such that [itex]x^2 = c[/itex], but that follows from the least upper bound principle using the same argument by which it is shown that there exists an [itex]x \in \mathbb{R}[/itex] such that [itex]x^2 = 2[/itex].)
 
  • #3
Ok, I really can't understand what you are telling me for the case of the semigroup. Both x and 1/x are negative; intuitively, I can see that because of continuity of real numbers, for a small lxl=x1 so that 1/-x1 belongs to the semigroup, I can take a smaller lxl=x2 as the average (x1+0)/2, so that 1/-x2 also belongs to the semigroup and is smaller (more negative) than 1/-x1. Can you please explain why did you suggest looking at 1/(2M)<0 ?

For the case of the function, I think I see what you tell me: it must exist x so that f(x)=M-1, proving that is not so hard, anyway I would have to review discriminants and quadratic equations for doing that.
 
  • #4
paalfis said:
Ok, I really can't understand what you are telling me for the case of the semigroup. Both x and 1/x are negative; intuitively, I can see that because of continuity of real numbers, for a small lxl=x1 so that 1/-x1 belongs to the semigroup, I can take a smaller lxl=x2 as the average (x1+0)/2, so that 1/-x2 also belongs to the semigroup and is smaller (more negative) than 1/-x1. Can you please explain why did you suggest looking at 1/(2M)<0 ?

Is [itex](1/(2M))^{-1} = 2M[/itex] less than or greater than [itex]M[/itex] when [itex]M < 0[/itex]?
 
  • #5
thanks!
 

FAQ: Proof for interval/function boundaries.

1. What is the definition of an interval in mathematics?

An interval is a connected subset of the real numbers that contains all the numbers between any two elements of the subset. It can be represented by using parentheses, brackets, or a combination of both.

2. How do you prove the boundaries of an interval or function?

To prove the boundaries of an interval or function, you can use various mathematical techniques such as the squeeze theorem, the intermediate value theorem, or the extreme value theorem. These theorems help to determine the maximum and minimum values of a function within a given interval.

3. Can you provide an example of proving interval boundaries using the squeeze theorem?

Yes, for example, let's say we want to find the limit of the function f(x) = x^2 as x approaches 2. By using the squeeze theorem, we can prove that the limit is 4. We can construct a lower and upper bound for the function by using the inequalities 2x < x^2 < 3x. As x approaches 2, the lower and upper bounds approach 4, thus proving that the limit is 4.

4. What is the importance of proving interval/function boundaries in mathematics?

Proving interval/function boundaries is important in mathematics as it helps to determine the behavior and properties of a function within a given interval. It also allows us to make predictions and draw conclusions about the function's behavior outside of the given interval.

5. Can interval/function boundaries ever be undefined?

Yes, interval/function boundaries can be undefined in certain cases. For example, if a function is discontinuous or has a vertical asymptote within the given interval, the boundaries will be undefined. In these cases, alternative methods such as using one-sided limits may be required to prove the boundaries.

Similar threads

Replies
5
Views
799
Replies
4
Views
530
Replies
8
Views
658
Replies
7
Views
756
Replies
17
Views
2K
Replies
11
Views
428
Back
Top